Exam 3 review questions

Pataasin ang iyong marka sa homework at exams ngayon gamit ang Quizwiz!

A 48-year-old policeman comes to your clinic, complaining of a swollen scrotum. He states it began a couple of weeks ago and has steadily worsened. He says the longer he stands up the worse it gets, but when he lies down it improves. He denies any pain with urination. Because he is impotent he doesn't know if intercourse would hurt. He states he has become more tired lately and has also gained 10 pounds in the last month. He denies any fever or weight loss. He has had some shortness of breath with exertion. His past medical history consists of type 2 diabetes for 20 years, high blood pressure, and coronary artery disease. He is on insulin, three high blood pressure pills, and a water pill. He has had his gallbladder removed. He is married and has five children. He is currently on disability because of his health problems. Both of his parents died of complications of diabetes. On examination you see a pleasant male appearing chronically ill. He is afebrile but his blood pressure is 160/100 and his pulse is 90. His head, eyes, ears, nose, throat, and neck examinations are normal. There are some crackles in the bases of each lung. During his cardiac examination there is an extra heart sound. Visualization of his penis shows an uncircumcised prepuce but no lesions or masses. Palpation of his scrotum shows generalized swelling, with no discrete masses. A gloved finger is placed through each inguinal ring, and with bearing down there are no bulges. The prostate is smooth and nontender. What abnormality of the scrotum is most likely the diagnosis? A) Hydrocele B) Scrotal hernia C) Scrotal edema D) Varicocele

Ans: C Feedback: Scrotal edema is a generalized swelling of the scrotum due to a systemic illness. No discrete masses are palpated. In this case, with the history of diabetes, hypertension, and coronary artery disease, the symptom of weight gain, and the signs of crackles in the lungs and an extra heart sound, the patient is probably suffering from congestive heart failure. This is also seen in patients with edema from hypoalbuminemia.

Which is a sign of benign prostatic hyperplasia? A) Weight loss B) Bone pain C) Fever D) Nocturia

Ans: D Feedback: Benign prostatic hyperplasia (BPH) is usually not associated with systemic symptoms such as weight loss or fever. Bone pain is associated with prostate cancer, which often metastasizes to the lower axial skeleton. Nocturia, sensation of incomplete voiding, weak stream, and difficulty initiating urination are also common symptoms of prostate cancer.

Mrs. Jaeger is a 67-year-old who went through menopause at age 55. She has now had some vaginal bleeding. Which of the following should be considered? A) Endometrial cancer B) Hormone replacement therapy C) Uterine or cervical polyps D) All of the above

Ans: D Feedback: Bleeding after menopause can have serious as well as benign causes. It is important to consider endometrial cancer as a cause of postmenopausal bleeding.

You are asked to perform a home safety assessment for an 87-year-old retired farmer who lives by himself. Which of the following is not considered to be an increased risk for falls? A) Loose electrical cords B) Slippery or irregular surfaces C) Chairs at awkward angles D) Bright lighting

Ans: D Feedback: Bright lighting is a recommendation to improve an older person's ability to see all possible things that could result in a fall.

An 18-year-old college freshman comes to your clinic, complaining of severe left-sided lower abdominal pain and a foul yellow discharge. The pain began last night while she was having intercourse with her boyfriend. Afterward, the pain became more severe and the discharge started. By this morning she had a fever of 101 degrees and walking was making the pain worse. Only lying very still makes the pain better. She has tried ibuprofen and acetaminophen without any improvement. She denies any nausea, vomiting, diarrhea, or constipation. Her past medical history is unremarkable. She has had two past sexual partners. She uses the birth control patch instead of condoms. She smokes a half pack of cigarettes a day and drinks four to five beers per weekend night. She denies any illegal drug use. Her parents are both healthy. On examination, you find a young woman who appears ill. Her temperature is 102 degrees and her pulse is elevated at 110. She is tender in the left lower quadrant but has no guarding or rebound. Speculum examination reveals yellow purulent drainage from the os. On palpation, there is cervical motion tenderness and the left adnexa is swollen and tender. A urine analysis is unremarkable and the urine pregnancy test is pending. What is the best choice of diagnosis for this adnexal swelling? A) Ovarian cyst B) Tubal pregnancy C) Pelvic inflammatory disease

Ans: C Feedback: PID is common in young sexually active woman and is usually caused by bacteria that have been sexually transmitted. It is often associated with fever, pelvic pain, and a purulent cervical discharge. On examination there is often cervical motion tenderness and adnexal swelling and pain. A purulent discharge is often seen in the cervical os. Causes of cervical infection are gonorrhea, Chlamydia, and sometimes herpes. This woman should be made aware that barrier methods of contraception may prevent transmission of these diseases, whereas the contraceptive patch or pill will not. It would be prudent to consider further history and screening for HIV in this patient.

Blood pressure abnormalities found more commonly in Western elderly include which of the following? A) Isolated elevation of the diastolic BP B) Narrow pulse pressure C) Elevation of the systolic BP D) Elevation of the BP with standing

Ans: C Feedback: Isolated systolic hypertension is common in the elderly because of stiffening of the large arteries. This is often accompanied by widening of the pulse pressure. Orthostatic BP changes are often seen with postural changes and can account for falls as well.

A 78-year-old retired seamstress comes to the office for a routine check-up. You obtain an ECG (electrocardiogram) because of her history of hypertension. You diagnose a previous myocardial infarction and ask her if she had any symptoms related to this. Which of the following symptoms would be more common in this patient's age group for an acute myocardial infarction? A) Chest pain B) Syncope C) Pain radiating into the left arm D) Pain radiating into the jaw

Ans: B Feedback: This is an atypical symptom and more likely to be seen in this patient's age group.

A patient comes to you for the appearance of red patches on his forearms that have been present for several months. They remain for several weeks. He denies a history of trauma. Which of the following is likely? A) Actinic keratoses B) Pseudoscars C) Actinic purpura D) Cherry angiomas

Ans: C Feedback: Actinic purpura is a common benign skin condition of the elderly, frequently involving the forearms. Pseudoscars are white patches and cherry angiomas are bright-red raised lesions usually found on the torso. Actinic keratoses are lesions resembling nevi, often with features which would be concerning if considering melanoma (review the ABCDEs of melanoma), but they produce a slightly greasy scale when scratched with a nail.

A 56-year-old female comes to your clinic, complaining of her left breast looking unusual. She says that for 2 months the angle of the nipple has changed direction. She does not do self-examinations, so she doesn't know if she has a lump. She has no history of weight loss, weight gain, fever, or night sweats. Her past medical history is significant for high blood pressure. She smokes two packs of cigarettes a day and has three to four drinks per weekend night. Her paternal aunt died of breast cancer in her forties. Her mother is healthy but her father died of prostate cancer. On examination you find a middle-aged woman appearing older than her stated age. Inspection of her left breast reveals a flattened nipple deviating toward the lateral side. On palpation the nipple feels thickened. Lateral to the areola you palpate a nontender 4-cm mass. The axilla contains several fixed nodes. The right breast and axilla examinations are unremarkable. What visible skin change of the breast does she have? A) Nipple retraction B) Paget's disease C) Peau d'orange sign

Ans: A Feedback: A retracted nipple is flattened or pulled inward or toward the medial, lateral, anterior, or posterior side of the breast. The surrounding skin can be thickened. This is a relatively late finding in breast cancer.

A 45-year-old African-American minister comes to your clinic for a general physical examination. He has not been feeling very well for about 3 months, including night sweats and a chronic low-grade fever of 100 to 101 degrees. He denies any upper respiratory symptoms, chest pain, nausea, constipation, diarrhea, blood in his stool, or urinary tract symptoms. He has had some lower back pain. He has a past history of difficult-to-control high blood pressure and high cholesterol. He has had no surgeries in the past. His mother has diabetes and high blood pressure. He knows very little about his father because his parents divorced when he was young. He knows his father died in his 50s, but he is unsure of the exact cause. The patient denies smoking, drinking, or drug use. He is married and has three children. On examination he appears his stated age and is generally fit. His temperature is 99.9 degrees and his blood pressure is 160/90. His head, ears, nose, throat, and neck examinations are normal. His cardiac, lung, and abdominal examinations are also normal. On visualization of the anus there is no inflammation, masses, or fissures. Digital rectal examination elicits an irregular, asymmetric, hard nodule on the otherwise normal posterior surface of the prostate. Examination of the scrotum and penis are normal. Laboratory results are pending. What disorder of the anus, rectum, or prostate is mostly likely in this case? A) Benign prostatic hyperplasia (BPH) B) Prostatitis C) Prostate cancer D) Anorectal cancer

Ans: C Feedback: Prostate cancer often presents with few symptoms and can sometimes be found on routine digital rectal examination. It is more common at a younger age in African-American men, and PSA screening, if indicated, begins at age 40, ten years earlier than in other races. Palpation on digital rectal examination can reveal a hard, irregular, asymmetric nodule, but can also reveal an asymmetry in texture between the two lobes.

A 16-year-old high school junior is brought to your clinic by his father. The teenager was taught in his health class at school to do monthly testicular self-examinations. Yesterday when he felt his left testicle it was enlarged and tender. He isn't sure if he has had burning with urination and he says he has never had sexual intercourse. He has had a sore throat, cough, and runny nose for the last 3 days. His past medical history is significant for a tonsillectomy as a small child. His father has high blood pressure and his mother is healthy. On examination you see a teenager in no acute distress. His temperature is 100.8 and his blood pressure and pulse are unremarkable. On visualization of his penis, he is uncircumcised and has no lesions or discharge. His scrotum is red and tense on the left and normal appearing on the right. Palpating his left testicle reveals a mildly sore swollen testicle. The right testicle is unremarkable. An examining finger is put through both inguinal rings, and there are no bulges with bearing down. His prostate examination is unremarkable. Urine analysis is also unremarkable. What abnormality of the testes does this teenager most likely have? A) Acute orchitis B) Acute epididymitis C) Torsion of the spermatic cord D) Prostatitis

Ans: A Feedback: Acute orchitis causes an inflamed, tender testicle. The scrotum will be red and tense. Orchitis is usually unilateral and often associated with viral infections such as mumps.

A 30-year-old man notices a firm, 2-cm mass under his areola. He has no other symptoms and no diagnosis of breast cancer in his first-degree relatives. What is the most likely diagnosis? A) Breast tissue B) Fibrocystic disease C) Breast cancer D) Lymph node

Ans: A Feedback: Approximately one third of adult men will have palpable breast tissue under the areola. While males can have breast cancer, this is much less common. There are no lymph nodes in this area.

A 60-year-old coach comes to your clinic, complaining of difficulty starting to urinate for the last several months. He believes the problem is steadily getting worse. When asked he says he has a very weak stream and it feels like it takes 10 minutes to empty his bladder. He also has the urge to go to the bathroom more often than he used to. He denies any blood or sediment in his urine and any pain with urination. He has had no fever, weight gain, weight loss, or night sweats. His medical history includes type 2 diabetes and high blood pressure treated with medications. He does not smoke but drinks a six pack of beer weekly. He has been married for 35 years. His mother died of a myocardial infarction in her 70s and his father is currently in his 80s with high blood pressure and arthritis. On examination you see a mildly obese male who is alert and cooperative. His blood pressure is 130/70 with a heart rate of 80. He is afebrile and his cardiac, lung, and abdominal examinations are normal. On visualization of the anus you see no inflammation, masses, or fissures. Digital rectal examination reveals a smooth, enlarged prostate. No discrete masses are felt. There is no blood on the glove or on guaiac testing. An analysis of the urine shows no red blood cells, white blood cells, or bacteria. What disorder of the anus, rectum, or prostate is this most likely to be? A) Benign prostatic hyperplasia (BPH) B) Prostatitis C) Prostate cancer D) Anorectal cancer

Ans: A Feedback: BPH becomes more prevalent during the fifth decade and is often associated with the urinary symptoms of hesitancy in starting a stream, decreased strength of stream, nocturia, and leaking of urine. On examination an enlarged, symmetric, firm prostate is palpated. The anterior lobe cannot be felt. These patients may also develop UTIs secondary to the obstruction.

A 24-year-old travel agent comes to your clinic, complaining of pain and swelling in her vulvar area. She states that 2 days earlier she could feel a small tender spot on the left side of her vagina but now it is larger and extremely tender. Her last period was 1 year ago and she is sexually active. She uses the Depo-Provera shot for contraception. She denies any nausea, vomiting, constipation, diarrhea, pain with urination, or fever. Her past medical history is significant for ankle surgery. Her mother is healthy and her father has type 2 diabetes. On examination, she appears her stated age and is standing up. She states she cannot sit down without excruciating pain. Her blood pressure, temperature, and pulse are unremarkable. On the visualization of her perineum, a large, red, tense swelling is seen to the left of her introitus. Palpation of the mass causes a great deal of pain. What disorder of the vulva is most likely causing her problems? A) Bartholin's gland infection B) Vulvar carcinoma C) Secondary syphilis D) Condylomata acuminata

Ans: A Feedback: Bartholin's gland infections cause a red-hot tender abscess at the duct opening to the Bartholin's glands. Gonococci, Chlamydia, and other organisms often cause them. Size is variable; if chronic, the infection can present as a nontender cyst.

A 22-year-old architecture major comes to your office, complaining of severe burning with urination, a fever of 101 degrees, and aching all over. She denies any upper respiratory, gastrointestinal, cardiac, or pulmonary symptoms. Her past medical history consists of severe acne. She is currently on an oral contraceptive. She has had no pregnancies or surgeries. She reports one new partner within the last month. She does not smoke but does drink occasionally. Her parents are both in good health. On examination you see a young woman appearing slightly ill. Her temperature is 100.3 and her pulse and blood pressure are unremarkable. Her head, ears, eyes, nose, throat, cardiac, pulmonary, and abdominal examinations are unremarkable. Palpation of the inguinal nodes shows lymphadenopathy bilaterally. On the visualization of the perineum, there are more than 10 shallow ulcers along each side of the vulva. Speculum and bimanual examination are unremarkable for findings, although she is very tender at the introitus. Urine analysis has some white blood cells but no red blood cells or bacteria. Her urine pregnancy test is negative. Which disorder of the vulva is most likely in this case? A) Genital herpes B) Condylomata acuminata C) Syphilitic chancre D) Epidermoid cyst

Ans: A Feedback: Genital herpes consists of small, shallow, painful ulcers. Primary infections are often associated with fever, malaise, and regional lymphadenopathy. The outbreak occurs generally between 1 and 3 weeks after exposure. Herpes is contagious and the majority of transmission occurs without the presence of obvious lesions. Transmission during passage through the birth canal can cause serious illness in affected newborns.

Which of the following is true regarding breast self-examination? A) It has been shown to reduce mortality from breast cancer. B) It is recommended unanimously by organizations making screening recommendations. C) A high proportion of breast masses are detected by breast self-examination. D) The undue fear caused by finding a mass justifies omitting instruction in breast self-examination.

Ans: C Feedback: Although self-examination has not been shown to reduce mortality and is not recommended by all groups making screening recommendations, many choose to teach women a systematic method in which to examine their breasts. A high proportion of breast masses are detected by breast self-examination.

A 40-year-old mother of two presents to your office for consultation. She is interested in knowing what her relative risks are for developing breast cancer. She is concerned because her sister had unilateral breast cancer 6 years ago at age 38. The patient reports on her history that she began having periods at age 11 and has been fairly regular ever since, except during her two pregnancies. Her first child arrived when she was 26 and her second at age 28. Otherwise she has had no health problems. Her father has high blood pressure. Her mother had unilateral breast cancer in her 70s. The patient denies tobacco, alcohol, or drug use. She is a family law attorney and is married. Her examination is essentially unremarkable. Which risk factor of her personal and family history most puts her in danger of getting breast cancer? A) First-degree relative with premenopausal breast cancer B) Age at menarche of less than 12 C) First live birth between the ages of 25 and 29 D) First-degree relative with postmenopausal breast cancer

Ans: A Feedback: Having a first-degree relative with cancer before menopause gives a relative risk of 3.1

A 72-year-old retired truck driver comes to the clinic with his wife for evaluation of hearing loss. He has noticed some decreased ability to hear what his wife and grandchildren are saying to him. He admits to lip-reading more. He has a history of noise exposure in his young adult years: He worked as a sound engineer at a local arena and had to attend a lot of concerts. Based on this information, what is the most likely finding regarding his hearing acuity? A) Loss of acuity for middle-range sounds B) Increase of acuity for low-range sounds C) Loss of acuity for high-range sounds D) Increase of acuity for high-range sounds

Ans: A Feedback: Human speech is considered to be a middle-range sound. During the aging process there is a loss of acuity, starting with high-pitched sounds but extending to the middle range and then into the low range.

Which of the following brief screening measures is useful in assessing memory? A) Three-item recall B) Serial 7s C) Spelling "world" backward D) Copying intersecting pentagrams

Ans: A Feedback: If the patient is unable to remember three items after 1 minute has passed, then this is a positive screening test and indicates a need for further testing. This is part of the "10-Minute Geriatric Screener."

Which of the following is true of assessment of the vascular system in the elderly? A) Fewer than one third of patients with peripheral vascular disease have symptoms of claudication. B) An aortic width of 2.5 cm is abnormal. C) Bruits are commonly benign findings. D) Orthostatic blood pressure and pulse are not useful in this population.

Ans: A Feedback: It is the minority of patients with peripheral vascular disease who experience claudication; therefore, ankle-brachial ratios should be performed more frequently. The aorta should be 3 cm or less. Bruits usually indicate pathology, and even when there is not a significant blockage, the risk of vascular disease throughout the body is increased. Orthostatic vital signs are very useful in this population. Remember to observe the pulse as well, as failure of the heart to increase its rate is a common cause of orthostatic hypotension. This can occur as a result of autonomic neuropathy or medications such as beta-blockers, among other causes.

A 63-year-old nurse comes to your office, upset because she has found an enlarged lymph node under her right arm. She states she found it last week while taking a shower. She isn't sure if she has any breast lumps because she doesn't know how to do self-exams. She states her last mammogram was 5 years ago and it was normal. Her past medical history is significant for high blood pressure and chronic obstructive pulmonary disease. She quit smoking 2 years ago after a 55-packs/year history. She denies using any illegal drugs and drinks alcohol rarely. Her mother died of a heart attack and her father died of a stroke. She has no children. On examination you see an older female appearing her stated age. On visual inspection of her right axilla you see nothing unusual. Palpating this area, you feel a 2-cm hard, fixed lymph node. She denies any tenderness. Visualization of both breasts is normal. Palpation of her left axilla and breast is unremarkable. On palpation of her right breast you feel a nontender 1-cm lump in the tail of Spence. What disorder of the axilla is most likely responsible for her symptoms? A) Breast cancer B) Lymphadenopathy of infectious origin C) Hidradenitis suppurativa

Ans: A Feedback: Metastatic lymph nodes tend to be hard, nontender, and fixed, often to the rib cage. Although the patient has no family history of breast cancer, she is at a slightly increased risk due to her never having had children.

A 19-year-old female comes to your office, complaining of a clear discharge from her right breast for 2 months. She states that she noticed it when she and her boyfriend were "messing around" and he squeezed her nipple. She continues to have this discharge anytime she squeezes that nipple. She denies any trauma to her breasts. Her past medical history is unremarkable. She denies any pregnancies. Both of her parents are healthy. She denies using tobacco or illegal drugs and drinks three to four beers a week. On examination her breasts are symmetric with no skin changes. You are able to express clear discharge from her right nipple. You feel no discrete masses and her axillae are normal. The remainder of her heart, lung, abdominal, and pelvic examinations are unremarkable. A urine pregnancy test is negative. What cause of nipple discharge is the most likely in her circumstance? A) Benign breast abnormality B) Breast cancer C) Nonpuerperal galactorrhea

Ans: A Feedback: Nipple discharge in benign breast abnormalities tends to be clear and unilateral. The discharge is usually not spontaneous. This patient needs to be told to stop compressing her nipple. If the problem still persists after the patient has stopped compressing the nipple, further workup is warranted.

Which of the following booster immunizations is recommended in the older adult population? A) Tetanus B) Diphtheria C) Measles D) Mumps

Ans: A Feedback: Older adults who have received the primary series of three tetanus immunizations should receive the single booster dose of tetanus immunization every 10 years.

Mr. White's son brings him in today because he notes that Mr. White has not been himself lately. He seems forgetful and has not taken care of himself as he normally does. He has reported falling twice at home to his son and has telephoned late at night because of insomnia. His blood pressure and diabetes have been difficult to control and his warfarin dosing has become more difficult. Which of the following should you suspect? A) Alzheimer's dementia B) Alcohol use C) Urinary tract infection D) Stroke

Ans: B Feedback: All of these answers are common diseases of the elderly and many have atypical presentations in this age group. The fact that his hypertension has become more difficult to control and his warfarin dosing is challenging to manage should lead you to consider that there is alcohol use. Further questioning, quantifying his use of alcohol, and application of the CAGE questionnaire may be useful.

A 23-year-old waitress comes to your clinic complaining of severe pelvic pain radiating to her right side. The pain began yesterday and is getting much worse. She has had no burning with urination and denies any recent sexual activity. She has no nausea, vomiting, constipation, diarrhea, fever, or vaginal discharge. Her last period was 3 to 4 weeks ago. Her past medical history consists of severe acne, depression, and mild obesity. She has had no surgeries. She broke up with her boyfriend 6 months ago and denies dating anyone else. She smokes one pack of cigarettes a day, drinks three to four beers two to three times a week, and denies any illegal drug use. Her mother is diabetic and her father has coronary artery disease. On examination you see a mildly obese female in moderate distress. Her blood pressure is 130/80 and her pulse is 90. She is afebrile. On auscultation she has active bowel sounds. She has no rebound or guarding in any abdominal quadrant. Speculum examination shows no lesions on the cervix and no discharge or bleeding from the os. During the bimanual examination, she has no cervical motion tenderness, but her right adnexal area is swollen and tender. A urine analysis is normal and the urine pregnancy test is pending. What disorder of the adnexa is most likely the diagnosis? A) Ovarian cyst B) Tubal pregnancy C) Pelvic inflammatory disease

Ans: A Feedback: Ovarian cysts often occur just before the onset of menses. They are also common in a disease known as polycystic ovarian syndrome. Other symptoms of this disorder are acne, hirsutism (increased hair growth), irregular periods, obesity. This disorder runs in families and later manifestations include diabetes, high blood pressure, and coronary artery disease. Single cysts on the right side can mimic the symptoms of appendicitis

An elderly woman with dementia is brought in by her daughter for a "rectal mass." On examination you notice a moist pink mass protruding from the anus, which is nontender. It is soft and does not have any associated bleeding. Which of the following is most likely? A) Rectal prolapse B) External hemorrhoid C) Perianal fistula D) Prolapsed internal hemorrhoid

Ans: A Feedback: Rectal prolapse is occasionally seen in chronic constipation. It represents actual rectal tissue which has protruded through the anus. In young children it is associated with cystic fibrosis as well as other conditions. An external hemorrhoid or a prolapsed internal hemorrhoid is not moist and does not have the same mucosa. A perianal fistula represents a connection from the bowel to the exterior apart from the anus and can be associated with inflammatory bowel disease, especially Crohn's disease.

When should a woman conduct breast self-examination with respect to her menses? A) Five to seven days following her menses B) Midcycle C) Immediately prior to menses D) During her menses

Ans: A Feedback: The breast examination should be conducted during the time with the least estrogen stimulation of the breast tissue. This corresponds to five to seven days following menses.

A 36-year-old security officer comes to your clinic, complaining of a painless mass in his scrotum. He found it 3 days ago during a testicular self-examination. He has had no burning with urination and no pain during sexual intercourse. He denies any weight loss, weight gain, fever, or night sweats. His past medical history is notable for high blood pressure. He is married and has three healthy children. He denies using illegal drugs, smokes two to three cigars a week, and drinks six to eight alcoholic beverages per week. His mother is in good health and his father had high blood pressure and coronary artery disease. On physical examination he appears anxious but in no pain. His vital signs are unremarkable. On visualization of his penis, he is circumcised and has no lesions. His inguinal region has no lymphadenopathy. Palpation of his scrotum shows a soft cystic-like lesion measuring 2 cm over his right testicle. There is no difficulty getting a gloved finger through either inguinal ring. With weight bearing there are no bulges. His prostate examination is unremarkable. What disorder of the scrotum does he most likely have? A) Hydrocele B) Scrotal hernia C) Testicular tumor D) Varicocele

Ans: A Feedback: The hydrocele is a fluid-filled cyst originating within the tunica vaginalis. An examining finger can be placed over the mass into the inguinal ring. An outside light source can be placed beneath the scrotum. Hydroceles often transilluminate light, whereas solid tumors do not.

Which of the following represents age-related changes in the lungs? A) Decrease in chest wall compliance B) Speed of expiration increases C) Increase in respiratory muscle strength D) Increased elastic recoil of lung tissue

Ans: A Feedback: The lungs age along with the rest of the body. These changes include decreased lung and chest wall compliance, increased expiratory time, decreased muscle strength and cough, and decreased elastic recoil.

Dawn is a 55-year-old woman who comes in today for her yearly wellness examination. You carefully perform the rectal examination in the lithotomy position and feel a mass against the bowel wall which is firm and immobile. Which of the following is most likely? A) Colon cancer B) Hemorrhoid C) Anal fissure D) Valve of Houston

Ans: A Feedback: This examination should make you think of colon cancer because the mass is firm, nonmobile, and nontender. Hemorrhoids are not firm and are frequently visible externally, although some may be internal as well. An anal fissure would be a palpable linear lesion in the anal canal that may be tender. Valves of Houston are sometimes palpable but are not firm.

Which of the following questions is part of the screening for physical disability? A) Are you able to go shopping for groceries or clothes? B) Are you able to walk one block? C) Are you able to pass the driver's license test? D) Are you able to perform light dusting and pick up after yourself around the house?

Ans: A Feedback: This is part of the Physical Disability screening portion of the 10-Minute Geriatric Screener.

Mrs. Buckley is a 75-year-old widow who wants you to look at her teeth because over the past 2 weeks she has had right-sided jaw pain when eating. It does not occur otherwise. She also has had a headache. Which of the following should be considered? A) Palpation of her temples B) Dental referral C) Ultrasound of the gallbladder D) Inquiry about anosmia

Ans: A Feedback: This story can be consistent with temporal arteritis, which can cause blindness in 15% of those affected. Early recognition is crucial. Most of these patients will have tenderness over one or both of the temporal arteries, and some have diminished temporal pulses as well. Early treatment with corticosteroids is indicated. It can also be associated with polymyalgia rheumatic, a condition which causes pain in the shoulder girdles and pelvis.

A 55-year-old retired property manager comes to your clinic, concerned that she may have a tumor in her rectum. When asked why she states that after straining at a bowel movement she felt a mass around her rectum. She denies any blood in her stool, black stools, or pain with defecation. She admits to having had chronic constipation for 30 years. She often uses laxatives to be able to have a bowel movement. She denies any recent weight gain, weight loss, fever, or night sweats. Her past medical history consists of hypothyroidism, and she has had two spontaneous vaginal deliveries. Her mother died recently of colon cancer and her father has high blood pressure but is otherwise healthy. She denies any smoking and only occasionally drinks alcohol. On examination, she seems nervous. Her blood pressure is 140/90 and her pulse is 100. Her cardiac, lung and abdominal examinations are normal. On the visualization of her anus, no inflammation, masses, or fissures are noted. When she is asked to bear down, you see a rosette of red mucosa prolapsing from the anus. On digital rectal examination, there are no masses and no blood is found on the glove. What disorder of the anus or rectum is this likely to be? A) Prolapse of the rectum B) Internal hemorrhoids C) Anorectal cancer D) Prostate cancer

Ans: A Feedback: Unless someone is bearing down, such as during a bowel movement, the red mucosa is not seen. This is common when there is heavy straining during a bowel movement. This finding in a young child or infant may lead you to suspect cystic fibrosis.

Induration along the ventral surface of the penis suggests which of the following? A) Urethral stricture B) Testicular carcinoma C) Peyronie's disease D) Epidermoid cysts

Ans: A Feedback: Urethral stricture may cause induration of the ventral surface of the penis. It more rarely represents a local carcinoma. A testicular carcinoma would be much more likely to occur in the scrotum. Peyronie's disease often causes induration on the dorsal proximal penis, and epidermoid cysts are benign findings on the scrotum.

A 20-year-old part-time college student comes to your clinic, complaining of growths on his penile shaft. They have been there for about 6 weeks and haven't gone away. In fact, he thinks there may be more now. He denies any pain with intercourse or urination. He has had three former partners and has been with his current girlfriend for 6 months. He says that because she is on the pill they don't use condoms. He denies any fever, weight loss, or night sweats. His past medical history is unremarkable. In addition to college, he works part-time for his father in construction. He is engaged to be married and has no children. His father is healthy and his mother has hypothyroidism. On examination the young man appears healthy. His vital signs are unremarkable. On visualization of his penis you see several moist papules along all sides of his penile shaft and even two on the corona. He has been circumcised. On palpation of his inguinal region there is no inguinal lymphadenopathy. Which abnormality of the penis does this patient most likely have? A) Condylomata acuminata B) Genital herpes C) Syphilitic chancre D) Penile carcinoma

Ans: A Feedback: Warts are generally painless papules along the shaft and corona. They are likely to spread and are caused by the human papilloma virus, transmitted through sexual contact. You should discuss prevention of STIs with him. Although his girlfriend's contraceptive pill protects her from pregnancy, he and she are unprotected from sharing STIs. She should receive regular Pap examinations and consider the HPV vaccine.

A 43-year-old store clerk comes to your office upset because she has found an enlarged lymph node under her left arm. She states she found it yesterday when she was feeling pain under her arm during movement. She states the lymph node is about an inch long and is very painful. She checks her breasts monthly and gets a yearly mammogram (her last was 2 months ago), and until now everything has been normal. She states she is so upset because her mother died in her 50s of breast cancer. The patient does not smoke, drink, or use illegal drugs. Her father is in good health. On examination you see a tense female appearing her stated age. On visual inspection of her left axilla you see a tense red area. There is no scarring around the axilla. Palpating this area, you feel a 2-cm tender, movable lymph node underlying hot skin. Other shotty nodes are also in the area. Visualization of both breasts is normal. Palpation of her right axilla and both breasts is unremarkable. Examining her left arm, you see a scabbed-over superficial laceration over her left hand. Upon your questioning, she remembers she cut her hand gardening last week. What disorder of the axilla is most likely responsible for her symptoms? A) Breast cancer B) Lymphadenopathy of infectious origin C) Hidradenitis suppurativa

Ans: B Feedback: A lymph node enlarged because of infection is generally hot, tender, and red. Close examination of the skin that drains to that lymph node region is advised. Often there will be a cut or scratch over the involved arm that has an infectious agent. An example is cat scratch disease.

Which is true of women who have had a unilateral mastectomy? A) They no longer require breast examination. B) They should be examined carefully along the surgical scar for masses. C) Lymphedema of the ipsilateral arm usually suggests recurrence of breast cancer. D) Women with breast reconstruction over their mastectomy site no longer require examination.

Ans: B Feedback: A woman who has had breast cancer remains at high risk for recurrence, especially in the contralateral breast. The mastectomy site should be carefully examined for local recurrence as well. Lymphedema or swelling of the ipsilateral arm following mastectomy is common and does not usually indicate recurrence. Women with breast reconstruction must also undergo careful examination.

A 23-year-old computer programmer comes to your office for an annual examination. She has recently become sexually active and wants to be placed on birth control. Her only complaint is that the skin in her armpits has become darker. She states it looks like dirt, and she scrubs her skin nightly with soap and water but the color stays. Her past medical symptoms consist of acne and mild obesity. Her periods have been irregular for 3 years. Her mother has type 2 diabetes and her father has high blood pressure. The patient denies using tobacco but has four to five drinks on Friday and Saturday nights. She denies any illegal drug use. On examination you see a mildly obese female who is breathing comfortably. Her vital signs are unremarkable. Looking under her axilla, you see dark, velvet-like skin. Her annual examination is otherwise unremarkable. What disorder of the breast or axilla is she most likely to have? A) Peau d'orange B) Acanthosis nigricans C) Hidradenitis suppurativa

Ans: B Feedback: Acanthosis nigricans can be associated with an internal malignancy, but in most cases it is a benign dermatologic condition associated with polycystic ovarian syndrome, consisting of acne, hirsutism, obesity, irregular periods, infertility, ovarian cysts, and early onset type 2 diabetes. It is also known to correlate with insulin resistance.

A 14-year-old junior high school student is brought in by his mother and father because he seems to be developing breasts. The mother is upset because she read on the Internet that smoking marijuana leads to breast enlargement in males. The young man adamantly denies using any tobacco, alcohol, or drugs. He has recently noticed changes in his penis, testicles, and pubic hair pattern. Otherwise, his past medical history is unremarkable. His parents are both in good health. He has two older brothers who never had this problem. On examination you see a mildly overweight teenager with enlarged breast tissue that is slightly tender on both sides. Otherwise his examination is normal. He is agreeable to taking a drug test. What is the most likely cause of his gynecomastia? A) Breast cancer B) Imbalance of hormones of puberty C) Drug use

Ans: B Feedback: Approximately one third of teenage boys develop gynecomastia during puberty. It is not surprising that the two older brothers did not have this.

Which of the following is most likely benign on breast examination? A) Dimpling of the skin resembling that of an orange B) One breast larger than the other C) One nipple inverted D) One breast with dimple when the patient leans forward

Ans: B Feedback: Asymmetry in size of the breasts is a common benign finding. The others are concerning for underlying malignancy.

A 51-year-old cook comes to your office for consultation. She recently found out that her 44-year-old sister with premenopausal breast cancer is positive for the BRCA1 gene. Your patient has been doing research on the Internet and saw that her chance of having also inherited the BRCA1 gene is 50%. She is interested in knowing what her risk of developing breast cancer would be if she were positive for the gene. She denies any lumps in her breasts and has had normal mammograms. She has had no weight loss, fever, or night sweats. Her mother is healthy and her father has prostate cancer. Two of her paternal aunts died of breast cancer. She is married. She denies using tobacco or illegal drugs and rarely drinks alcohol. Her breast and axilla examinations are unremarkable. At her age, what is her risk of getting breast cancer if she has the BRCA1 gene? A) 10% B) 50% C) 80%

Ans: B Feedback: At the age of 50, the risk of breast cancer for someone with the BRCA1 gene is 50%.

A 73-year-old retired accountant comes to your office for her annual examination. She has incontinence of urine when she coughs or sneezes. She takes several medications for control of hypertension and diabetes. You use the DIAPERS mnemonic to assess the cause of her incontinence. All of the following are items represented by the mnemonic except for: A) Atrophic vaginitis B) Depression C) Pharmaceuticals D) Restricted mobility

Ans: B Feedback: Depression is not a risk factor for incontinence. The D in the mnemonic stands for delirium.

A 22-year-old unemployed roofer presents to your clinic, complaining of pain in his testicle and penis. He states the pain began last night and has steadily become worse. He states it hurts when he urinates and he has not attempted intercourse since the pain began. He has tried Tylenol and ibuprofen without improvement. He denies any fever or night sweats. His past medical history is unremarkable. He has had four previous sexual partners and has had a new partner for the last month. She is on oral contraceptives so he has not used condoms. His parents are both in good health. On examination you see a young man lying on his side. He appears mildly ill. His temperature is 100.2 and his blood pressure, respirations, and pulse are normal. On visualization of the penis he is circumcised, with no lesions or discharge from the meatus. Visualization of the scrotal skin appears unremarkable. Palpation of the testes shows severe tenderness at the superior pole of the normal-sized left testicle. He also has tenderness when you palpate the structures superior to the testicle through the scrotal wall. The right testicle is unremarkable. An examining finger is placed through each inguinal ring without bulges being noted with bearing down. His prostate examination is unremarkable. Urine analysis shows white blood cells and bacteria. What diagnosis of the male genitalia is most likely in this case? A) Acute orchitis B) Acute epididymitis C) Torsion of the spermatic cord D) Prostatitis

Ans: B Feedback: Epididymitis is an infection of the epididymis superior to the testicle. It can often be caused by sexually transmitted disease and can cause burning with urination and scrotal pain. Palpate the spermatic cord through the scrotum by pinching medially and sliding your pinched fingers laterally. The spermatic cord, including the epididymis, will pass between your fingers and be tender if involved.

Which of the following is true of human papilloma virus (HPV) infection? A) Pap smear is a relatively ineffective screening method. B) It commonly resolves spontaneously in 1-2 years. C) It is the second most common STI in the United States. D) HPV infections cause a small but important number of cervical cancers.

Ans: B Feedback: HPV is the most common STI in the United States and is by far the most common cause of cervical cancers. The sensitivity of the liquid-based cytology is between 61% and 95% and specificity is from 78% to 82%. While HPV affects almost 50% of the population at some point, many of these infections resolve spontaneously.

A 72-year-old retired saleswoman comes to your office, complaining of a bloody discharge from her left breast for 3 months. She denies any trauma to her breast. Her past medical history includes high blood pressure and abdominal surgery for colon cancer. Her aunt died of ovarian cancer and her father died of colon cancer. Her mother died of a stroke. The patient denies tobacco, alcohol, or drug use. She is a widow and has three healthy children. On examination her breasts are symmetric, with no skin changes. You are able to express bloody discharge from her left nipple. You feel no discrete masses, but her left axilla has a hard, 1-cm fixed node. The remainder of her heart, lung, abdominal, and pelvic examinations are unremarkable. What cause of nipple discharge is the most likely in her circumstance? A) Benign breast abnormality B) Breast cancer C) Galactorrhea

Ans: B Feedback: Nipple discharge in breast cancer is usually unilateral and can be clear or bloody. Although a breast mass is not palpated, in this case a fixed lymph node is palpated. Other forms of breast cancer can present as a chronic rash on the breast.

Jean has just given birth 6 months ago and is breast-feeding her child. She has not had a period since giving birth. What does this most likely represent? A) Primary amenorrhea B) Secondary amenorrhea C) Oligomenorrhea D) Dysmenorrhea

Ans: B Feedback: Periods will normally stop after menarche for several reasons, including pregnancy, lactation, and menopause. Failure to start periods usually indicates an endocrine problem and is referred to as primary amenorrhea. Oligomenorrhea represents infrequent menses and dysmenorrhea is pain with menstruation.

A 24-year-old graduate student comes to your clinic, complaining of burning during urination and increased urinary frequency. He has had a low-grade fever (100.5 degrees) and does not feel very well. He is very worried about sexually transmitted diseases because he had a drunken encounter 2 weeks ago and did not use a condom. He has had no recent weight loss, weight gain, or night sweats. His past medical history includes knee surgery in high school and genital warts in college. He does not smoke but drinks six beers every Friday and Saturday night. He denies using any IV drugs but has tried marijuana in the past. His father has high cholesterol but his mother is healthy. On examination, he appears tired. His temperature is 99.5 degrees and his blood pressure is 110/70. His abdominal examination is normal. Visualization of the anus shows no masses, inflammation, or fissures. Digital rectal examination reveals a warm, boggy, tender prostate. No discrete masses are felt and there is no blood on the glove. The scrotum and penis appear normal. Urinalysis shows moderate amounts of white blood cells and bacteria. What disorder of the anus, prostate, or rectum best describes this situation? A) Benign prostatic hyperplasia (BPH) B) Prostatitis C) Prostate cancer D) Epididymitis

Ans: B Feedback: Prostatitis generally causes increased frequency of urination, pain with urination, and lower back pain. On digital rectal examination a warm, tender, boggy prostate will be palpated. In young men the etiology is often a sexually transmitted disease such as chlamydia or gonorrhea. This man's substance abuse problem should also be discussed with him, and you should consider further questions and screening for HIV.

A 29-year-old married computer programmer comes to your clinic, complaining of "something strange" going on in his scrotum. Last month while he was doing his testicular self-examination he felt a lump in his left testis. He waited a month and felt the area again, but the lump was still there. He has had some aching in his left testis but denies any pain with urination or sexual intercourse. He denies any fever, malaise, or night sweats. His past medical history consists of groin surgery when he was a baby and a tonsillectomy as a teenager. He eats a healthy diet and works out at the gym five times a week. He denies any tobacco or illegal drugs and drinks alcohol occasionally. His parents are both healthy. On examination, you see a muscular, healthy, young-appearing man with unremarkable vital signs. On visualization the penis is circumcised with no lesions; there is a scar in his right inguinal region. There is no lymphadenopathy. Palpation of his scrotum is unremarkable on the right but indicates a large mass on the left. Placing a finger through the inguinal ring on the right, you have the patient bear down. Nothing is felt. You attempt to place your finger through the left inguinal ring but cannot get above the mass. On rectal examination his prostate is unremarkable. What disorder of the testes is most likely the diagnosis? A) Hydrocele B) Scrotal hernia C) Scrotal edema D) Varicocele

Ans: B Feedback: Scrotal hernias occur when the small intestine passes through a weak spot of the inguinal ring. The examiner cannot get a finger above the hernia into the ring. Hernias are often caused by increased abdominal pressure, such as in weight lifting. Patients who have a hernia on one side often have another hernia on the opposite side. In this patient's case, a right-sided hernia was repaired as an infant.

A 42-year-old house painter comes to your clinic, complaining of pain with defecation and profuse bleeding in the toilet after a bowel movement. He was in his usual state of health until 2 weeks ago, when he was injured in a car accident. After the accident he began taking prescription narcotics for the pain in his shoulder. Since then he has had very few bowel movements. His stool is hard and pebble-like. He states he has always been "regular" in the past, with easy bowel movements. His diet has not changed but he states that he is exercising less since the accident. His past medical history includes hypertension and he is on a low-dose diuretic. He has had no other chronic illnesses or surgeries. He has a family history of hypertension, coronary heart disease, and diabetes but no cancer. He is divorced and has three children. He smokes two packs of cigarettes per day and quit drinking more than 10 years ago. He has had no recent weight loss, weight gain, fever, or night sweats. On examination he appears muscular and healthy; he is afebrile. His blood pressure is 135/90 with a pulse of 80. His cardiac, lung, and abdominal examinations are normal. He is wearing a sling on his left arm. On observation of his anus you find a swollen bluish ovoid mass that appears to contain a blood clot. Digital rectal examination is extremely painful for the patient. No other mass is palpated within the anus or rectum. What disorder of the anus is this patient likely to have? A) Anal fissure B) External hemorrhoid C) Anorectal cancer D) Internal hemorrhoid

Ans: B Feedback: Swollen, bluish ovoid mass is most likely a thrombosed external hemorrhoid. These can cause brisk bleeding with defecation. Hemorrhoids are often caused by low-fiber diets, dehydration, lack of exercise, and anything that causes constipation leading to increased straining with defecation. Narcotics can cause severe constipation, leading to this disorder.

Jim is a 47-year-old man who is having difficulties with sexual function. He is recently separated from his wife of 20 years. He notes that he has early morning erections but otherwise cannot function. Which of the following is a likely cause for his problem? A) Decreased testosterone levels B) Psychological issues C) Abnormal hypogastric arterial circulation D) Impaired neural innervation

Ans: B Feedback: The fact that he has an early morning erection is indicative of normal physiologic function. You may consider looking further into psychological issues, perhaps related to his marital difficulties. If the patient is unsure of whether early morning erections are occurring, some recommend the postage stamp test in which a ring of postage stamps or other perforated stickers is placed around the penis while in the flaccid state. If the perforations are broken, it is likely an erection has occurred. Do not perform this test without perforations in the stickers, or the ring may function as a tourniquet.

Frank is a 24-year-old man who presents with multiple burning erosions on the shaft of his penis and some tender inguinal adenopathy. Which of the following is most likely? A) Primary syphilis B) Herpes simplex C) Chancroid D) Gonorrhea

Ans: B Feedback: The multiplicity of lesions as well as the burning quality of the pain would lead one to suspect herpes simplex. Syphilis usually presents with a single chancre which is generally painless. Chancroid forms a single, jagged, deep ulcer and gonorrhea usually results in a burning discharge without skin lesions.

Which of the following conditions involves a tight prepuce which, once retracted, cannot be returned? A) Phimosis B) Paraphimosis C) Balanitis

Ans: B Feedback: This describes paraphimosis. Phimosis describes a foreskin which cannot be retracted. Balanitis involves an inflammation of the glans, whereas balanoposthitis involves inflammation of both the glans and the prepuce.

Mr. Chin is an 82-year-old man who comes to your office for a routine check. On examination, you notice a somewhat high-pitched murmur in the second right intercostal space during systole. It does not radiate and the rest of his examination is normal for his age. Which is true of the most likely cause of this murmur? A) It often decreases carotid upstroke. B) It carries with it an increased risk for cardiovascular disease. C) It is usually accompanied by an S3 gallop. D) It is found in 10% of otherwise normal elderly patients.

Ans: B Feedback: This murmur most likely represents aortic sclerosis, a common murmur affecting about one third of those near 60 years of age. It is caused by calcification of the valve and is associated with cardiovascular risk. Aortic sclerosis does not usually cause obstruction to normal flow, so carotid upstroke should be normal, and it is not associated with an S3 gallop.

A 44-year-old female comes to your clinic, complaining of severe dry skin in the area over her right nipple. She denies any trauma to the area. She noticed the skin change during a self-examination 2 months ago. She also admits that she had felt a lump under the nipple but kept putting off making an appointment. She does admit to 6 months of fatigue but no weight loss, weight gain, fever, or night sweats. Her past medical history is significant for hypothyroidism. She does not have a history of eczema or allergies. She denies any tobacco, alcohol, or drug use. On examination you find a middle-aged woman appearing her stated age. Inspection of her right breast reveals a scaly eczema-like crust around her nipple. Underneath you palpate a nontender 2-cm mass. The axilla contains only soft, moveable nodes. The left breast and axilla examination findings are unremarkable. What visible skin change of the breast does she have? A) Nipple retraction B) Paget's disease C) Peau d'orange sign

Ans: B Feedback: This uncommon form of breast cancer starts as an eczema-like, scaly skin change around the areola. The lesion may weep, crust, or erode. It can be associated with an underlying mass, but the skin change can also be found alone. Any eczema-like area around the nipple that does not respond to topical treatment needs to be evaluated for breast cancer.

A 34-year-old married daycare worker comes to your office, complaining of severe pelvic pain for the last 6 hours. She states that the pain was at first cramp-like but is now sharp. Nothing makes the pain better or worse. She has had no vaginal bleeding or discharge. She has had no pain with urination. She has had some nausea for the last few days but denies vomiting, constipation, or diarrhea. She states she feels so bad that when she stands up she has fainted. Her past medical history consists of two prior cesarean sections and an appendectomy. She is married and has two children. She denies any tobacco, alcohol, or drug use. Her parents are both healthy. On examination, you find a pale young woman who is obviously in a great deal of pain. She is lying on her right side with her eyes closed. Her blood pressure is 90/60 and her pulse is 110. She is afebrile. She has bowel sounds and her abdomen is soft. The speculum examination reveals a bluish cervix but no blood or purulent discharge at the os. There is a mild amount of tenderness with palpation of the cervix. The uterus is nongravid but the right adnexal area is swollen and very tender. Urine analysis is normal and the urine pregnancy test is pending. What type of adnexal disorder is causing her pain? A) Ovarian cysts B) Tubal pregnancy C) Pelvic inflammatory disease

Ans: B Feedback: Tubal pregnancies start to cause pain as the fetus grows too large to be contained in the tube. Eventually the tube begins to rupture and bleeding ensues, leading to hypotension, tachycardia, and syncope. On visualization of the cervix, the purple to bluish color of pregnancy may be seen.

A young man feels something in his scrotum and comes to you for clarification. On your examination, you note what feels like a "bag of worms" in the left scrotum, superior to the testicles. Which of the following is most likely? A) Hydrocele of the spermatic cord B) Varicocele C) Testicular carcinoma D) A normal vas deferens

Ans: B Feedback: Varicoceles are common in normal men. They are often found in the left scrotum or bilaterally and should normally resolve in the supine position. This is because they represent varicosities within the scrotum. These require further investigation if they occur only on the right side or do not resolve in the supine position. They can contribute to infertility because the testicles are unable to achieve a cool enough temperature for sperm production, due to increased blood flow from the varicocele. A hydrocele would be a painless mass on the spermatic cord and the vas deferens is palpated as part of the spermatic cord. You should lightly pinch the scrotum medially and move laterally until you feel the spermatic cord pass between your fingers.

A 48-year-old high school librarian comes to your clinic, complaining of 1 week of heavy discharge causing severe itching. She is not presently sexually active and has had no burning with urination. The symptoms started several days after her last period. She just finished a course of antibiotics for a sinus infection. Her past medical history consists of type 2 diabetes and high blood pressure. She is widowed and has three children. She denies tobacco, alcohol, or drug use. Her mother has high blood pressure and her father died of diabetes complications. On examination, you see a healthy-appearing woman. Her blood pressure is 130/80 and her pulse is 70. Her head, eyes, ears, nose, throat, cardiac, lung, and abdominal examinations are unremarkable. Palpation of the inguinal lymph nodes is unremarkable. On the visualization of the vulva, a thick, white, curdy discharge is seen at the introitus. On speculum examination, there is a copious amount of this discharge. The pH of the discharge is 4.1 and the KOH whiff test is negative, with no unusual smell. Wet prep shows budding hyphae. What vaginitis does this patient most likely have? A) Trichomonas vaginitis B) Candida vaginitis C) Bacterial vaginosis D) Atrophic vaginitis

Ans: B Feedback: Candida is associated with a thick, white, curd-like discharge that causes severe pruritus. The pH will be normal (£4.5) and the KOH whiff test will be normal. The wet prep often shows yeast spores and budding hyphae. Candida is very common in diabetics and after recent use of antibiotics. It is not thought to be sexually transmitted.

Which of the following would lead you to suspect a hydrocele versus other causes of scrotal swelling? A) The presence of bowel sounds in the scrotum B) Being unable to palpate superior to the mass C) A positive transillumination test D) Normal thickness of the skin of the scrotum

Ans: C Feedback: A cystic structure will often transilluminate well. While a transilluminator head for your battery handle is ideal, it is possible to use an otoscope to transilluminate the scrotum. You should be able to get above the mass on palpation and bowel sounds should not be present. If they are, it should lead you to consider an inguinal hernia. Scrotal edema involves thickened skin which can be measured by gently pinching a section of the scrotum itself.

A 36-year-old married bank teller comes to your office, complaining of pain with defecation and occasional blood on the toilet paper. She states that last week she had food poisoning with nausea, vomiting, and diarrhea. She had runny stools but no black or bloody stools. Ever since her illness, she has continued to have severe pain with bowel movements. She now tries to put off defecation as long as possible. Although she is having constipation she denies any further diarrhea or leakage of stool. She has a past medical history of hypothyroidism and two spontaneous vaginal deliveries. She has had no other chronic illnesses or surgeries. She does not smoke and rarely drinks. She has two children. There is no family history of breast or colon cancer. She has had no weight gain, weight loss, fever, or night sweats. On examination she is afebrile, with a blood pressure of 115/70 and a pulse of 80. On abdominal examination she has active bowel sounds, is nontender in all quadrants, and has no hepatosplenomegaly. Inspection of the anus reveals inflammation on the posterior side with erythema. Digital rectal examination is painful for the patient but no abnormalities are palpated. Anoscopic examination reveals no inflammation or bleeding. What is the anal disorder that best describes her symptoms? A) Anorectal fistula B) External hemorrhoids C) Anal fissure D) Anorectal cancer

Ans: C Feedback: Anal fissures often occur after severe diarrhea or constipation. They cause bright blood on the toilet paper and are extremely painful during defecation. A small ulceration or fissure is observed proximal to the anus.

You are examining an elderly man and notice the following: decreased vibration sense in the feet and ankles, diminished gag reflex, right patellar reflex less than the left, and diminished abdominal reflexes. Which of these is abnormal? A) Decreased vibration sense B) Diminished gag reflex C) Diminished right patellar reflex compared to the left D) Diminished abdominal reflexes

Ans: C Feedback: Asymmetry on any examination is usually reason for concern. The other changes are commonly associated with aging. You may consider looking for other neurologic signs on the right, although occasionally you may mistake an abnormally brisk reflex to be normal when compared to the other side. It is usually a good idea to question whether the opposite side is actually the abnormal one when you find asymmetry on examination.

A 30-year-old paralegal analyst comes to your clinic, complaining of a bad-smelling vaginal discharge with some mild itching, present for about 3 weeks. She tried douching but it did not help. She has had no pain with urination or with sexual intercourse. She has noticed the smell increased after intercourse and during her period last week. She denies any upper respiratory, gastrointestinal, cardiac, or pulmonary symptoms. Her past medical history consists of one spontaneous vaginal delivery. She is married and has one child. She denies tobacco, alcohol, or drug use. Her mother has high blood pressure and her father died from a heart disease. On examination, she appears healthy and has unremarkable vital signs. On examination of the perineum, there are no lesions noted. On palpation of the inguinal nodes, there is no lymphadenopathy. On speculum examination, a thin gray-white discharge is seen in the vault. The pH of the discharge is over 4.5 and there is a fishy odor when potassium hydroxide (KOH) is applied to the vaginal secretions on the slide. Wet prep shows epithelial cells with stippled borders (clue cells). What type of vaginitis best describes her findings? A) Trichomonas vaginitis B) Candida vaginitis C) Bacterial vaginosis D) Atrophic vaginitis

Ans: C Feedback: Bacterial vaginosis generally has a homogenous, grayish-white, thin discharge. The pH will be over 4.5 and the KOH wet prep releases a strong fishy odor, known as a "positive whiff test." Any basic pH fluid (semen or blood) will cause the fish-like odor to occur, often after intercourse, as with this patient. The wet prep will show clue cells, which are epithelial cells with borders stippled by bacteria.

A 28-year-old married clothing sales clerk comes to your clinic for her annual examination. She requests a refill on her birth control pills. Her only complaint is painless bleeding after intercourse. She denies any other symptoms. Her past medical history consists of two spontaneous vaginal deliveries. Her past six Pap smears have all been normal. She is married and has two children. Her mother is in good health and her father has high blood pressure. On examination you see a young woman appearing healthy and relaxed. Her vital signs are unremarkable and her head, eyes, ears, throat, neck, cardiac, lung, and abdominal examinations are normal. Visualization of the perineum shows no lesions or masses. Speculum examination shows a red mass at the os. On taking a Pap smear the mass bleeds easily. Bimanual examination shows no cervical motion tenderness and both ovaries are palpated and nontender. What is the most likely diagnosis for the abnormality of her cervix? A) Carcinoma of the cervix B) Mucopurulent cervicitis C) Cervical polyp D) Retention cyst

Ans: C Feedback: Cervical polyps are polyps of endometrial cells arising from either the uterus or the cervix. They are benign and usually painless but can bleed during intercourse.

Mrs. Glynn is 90 years old and lives alone. She is able to bathe, dress, prepare her food, and transfer from bed to chair independently. She has children in the area who help her with her medications and transportation needs. Which of the following is considered an instrumental activity of daily living? A) Bathing B) Dressing C) Preparing food D) Transferring from bed to chair

Ans: C Feedback: Instrumental activities of daily living involve higher thought processes such as preparing food, whereas bathing, dressing, and transferring are considered physical activities of daily living.

A 22-year-old nurse comes to your clinic, complaining of severe constipation and pain during defecation. She has also seen blood on the toilet paper. She states that she eats a healthy diet and does some light exercising. She is currently at the beginning of her third trimester of an unremarkable pregnancy. Her past medical history is unremarkable. Her mother has high cholesterol but her father is in good health. She does not smoke, drinks alcohol, or use illegal drugs. She is married and expecting her first child. On examination, she appears healthy and is afebrile, with a blood pressure of 110/60. Her abdominal examination reveals a gravid uterus but is otherwise unremarkable. On the visualization of the anus, there is a slight red, moist-appearing protrusion from the anus. As you have her bear down, the protrusion grows larger. On digital rectal examination, you can feel an enlarged tender area on the posterior side. There is some blood on the glove after the examination. What disorder of the anus or rectum best fits this presentation? A) Anal fissure B) External hemorrhoids C) Internal hemorrhoids D) Anorectal fistula

Ans: C Feedback: Internal hemorrhoids are common during pregnancy. A red, swollen, moist mass is seen prolapsing through the anus, which worsens with bearing down. These are not usually palpable on rectal examination if not prolapsed.

Mr. Jackson is a 50-year-old African-American who has had discomfort between his scrotum and anus. He also has had some fevers and dysuria. Your rectal examination is halted by tenderness anteriorly, but no frank mass is palpable. What is your most likely diagnosis? A) Prostate cancer B) Colon cancer C) Prostatitis D) Colonic polyp

Ans: C Feedback: The above examination, associated with a history of dysuria, frequency, and incomplete voiding, should lead you to suspect acute prostatitis. Prostate cancer, colon cancer, and polyps should not ordinarily cause systemic symptoms such as fever.

A 56-year-old homosexual man presents with itching, anorectal pain, and tenesmus of 1 week's duration. Rectal examination reveals generalized tenderness without frank prostate abnormalities. Which of the following is most likely? A) Acute prostatitis B) External hemorrhoid C) Proctitis D) Colon cancer

Ans: C Feedback: The combination of itching, anorectal pain, and tenesmus in a homosexual man should make one consider proctitis. This may be caused by a sexually transmitted infection such as gonorrhea, chlamydia, or lymphogranuloma venereum. A careful history should be taken, and counseling regarding protection from these diseases should be offered. While pain and itching are associated with hemorrhoids, the internal tenderness on examination makes this less likely than proctitis in this patient. Acute prostatitis does not usually cause itching and is usually associated with examination findings. Most colon cancers do not cause any symptoms, which is why screening for asymptomatic disease is so important.

How often, according to American Cancer Society recommendations, should a woman undergo a screening breast examination by a skilled clinician? A) Every year B) Every 2 years C) Every 3 years D) Every 4 years

Ans: C Feedback: The current recommendation for screening by breast examination is every 3 years.

Mrs. Geller is somewhat quiet today. She has several bruises of different colors on the ulnar aspects of her forearms and on her abdomen. She otherwise has no complaints and her diabetes and hypertension are well managed. Her son from out of state accompanies her today and has recently moved in to help her. What should you suspect? A) Overuse of aspirin B) Frequent falls C) Elder abuse D) Depression

Ans: C Feedback: The different colors of the bruising indicate that they have occurred at different times and are unlikely to have resulted from a single fall. The location of the bruising on the ulnar aspects of the forearms potentially indicates that she was trying to defend herself and are not typical areas to be bruised by a fall. Depression may be evident, but this is more likely to be a result rather than a cause of her situation today. While nothing is proven, it would be wise to interview her without her son in the room. If in doubt, a social worker consult may be helpful to determine if elder abuse is occurring.

Which is true of the pectinate or dentate line? A) It is a palpable landmark. B) It demarcates the areas supplied by the central nervous system from the peripheral nervous system. C) It is the border between the anal canal and the rectum. D) It is not visible on proctoscopic examination.

Ans: C Feedback: The pectinate or dentate line marks the division between the anal canal and rectum. It is not palpable but is visible on proctoscopy. It also marks the areas supplied by the visceral and peripheral nervous systems.

Which is the most effective pattern of palpation for breast cancer? A) Beginning at the nipple, make an ever-enlarging spiral. B) Divide the breast into quadrants and inspect each systematically. C) Examine in lines resembling the back and forth pattern of mowing a lawn. D) Beginning at the nipple, palpate outward in a stripe pattern.

Ans: C Feedback: The vertical strip pattern has been shown to be the most effective pattern for palpation of the breast. The most important aspect, however, is to be systematic. The tail of Spence, located on the upper anterior chest, is an area commonly missed on examination.

It is summer and an 82-year-old woman is brought to you from her home after seeing her primary care doctor 2 days ago. She was started on an antibiotic at that time. Today, she comes to the emergency room not knowing where she is or what year it is. What could be a likely cause of this? A) Alzheimer's dementia B) Stroke C) Delirium D) Meningitis

Ans: C Feedback: These are not signs of normal aging and seem to be of acute onset. This makes Alzheimer's less likely. Stroke and meningitis could cause these symptoms as well, but the combination of the heat and a recent infection make delirium much more likely. Though she was prescribed an antibiotic, she may not have improved because of bacterial resistance or because of noncompliance due to cost, depression, or even an underlying mild dementia. Dementia should not result in an acute mental status change, although illness may cause a worsening of dementia.

An 88-year-old retired piano teacher comes for evaluation of fatigue. You notice that her clothes are hanging loosely off her frame and that she has lost 15 pounds. She is unaware of this. Her husband of 63 years died a few months ago. You ask the patient to complete a Rapid Screen for Dietary Intake. Which of the following statements is considered to be part of this rapid screen? A) I eat more than two meals per day. B) I drink one glass of alcohol every day. C) Without wanting to, I have lost or gained 10 pounds in the last 6 months. D) I eat with at least one other person most of the time.

Ans: C Feedback: This is part of the Rapid Screen for Dietary Intake.

A 79-year-old retired banker comes to your office for evaluation of difficulty with urination; he gets up five to six times per night to urinate and has to go at least that often in the daytime. He does not feel as if his bladder empties completely; the strength of the urinary stream is diminished. He denies dysuria or hematuria. This problem has been present for several years but has worsened over the last 8 months. You palpate his prostate. What is your expected physical examination finding, based on this description? A) Normal size, smooth B) Normal size, boggy C) Enlarged size, smooth D) Enlarged size, boggy

Ans: C Feedback: This is the expected physical examination finding in benign prostatic hyperplasia (BPH).

A patient is concerned about a dark skin lesion on her anterolateral abdomen. It has not changed, and there is no discharge or bleeding. On examination, there is a medium brown circular lesion on the anterolateral wall of the abdomen. It is soft, has regular borders, is evenly pigmented, and is about 7 mm in diameter. What is this lesion? A) Melanoma B) Dysplastic nevus C) Supernumerary nipple D) Dermatofibroma

Ans: C Feedback: This represents a supernumerary nipple. These occur along the "milk line" and do not exhibit features of more concerning lesions.

Mr. Kelly comes to you today for a burning pain in his lower abdomen. This has gone on for 2 months. He has received radiation for prostatic cancer for the past quarter. What assumptions could you draw from this? A) This represents persistent pain. B) His pain reporting is likely to be unreliable. C) There are "red flags" present. D) He is depressed.

Ans: C Feedback: This scenario is consistent with acute pain, although this may become persistent if it lasts beyond 3 months. The burning quality to the pain should be a red flag, along with descriptions of pain as "discomfort" or "soreness." Depressed affect or changes in posture or gait are also red flags. Studies have found that pain reporting in the elderly is accurate. Although depression may be present, we have no indications of it in this scenario.

A 15-year-old high school football player is brought to your office by his mother. He is complaining of severe testicular pain since exactly 8:00 this morning. He denies any sexual activity and states that he hurts so bad he can't even urinate. He is nauseated and is throwing up. He denies any recent illness or fever. His past medical history is unremarkable. He denies any tobacco, alcohol, or drug use. His parents are both in good health. On examination you see a young teenager lying on the bed with an emesis basin. He is very uncomfortable and keeps shifting his position. His blood pressure is 150/100, his pulse is 110, and his respirations are 24. On visualization of the penis he is circumcised and there are no lesions and no discharge from the meatus. His scrotal skin is tense and red. Palpation of the left testicle causes severe pain and the patient begins to cry. His prostate examination is unremarkable. His cremasteric reflex is absent on the left but is normal on the right. By catheter you get a urine sample and the analysis is unremarkable. You send the boy with his mother to the emergency room for further workup. What is the most likely diagnosis for this young man's symptoms? A) Acute orchitis B) Acute epididymitis C) Torsion of the spermatic cord D) Prostatitis

Ans: C Feedback: Torsion is caused by the twisting of the testicle on its spermatic cord and blood vessels, leading to severe pain. The scrotum becomes red and tense. Torsion is usually seen in adolescents and is a true surgical emergency. If not quickly surgically repaired, the testicle's function is lost and it has to be removed. The presence of a cremasteric reflex is reassuring, but in this case a thorough evaluation must take place as soon as possible.

Mrs. Stanton is a 79-year-old widow who presents to your office for a routine BP visit. You note a new pulsatile mass in the right neck at the carotid artery. Which of the following is the most likely cause for this? A) Anxiety B) Carotid artery aneurysm C) Kinking of the artery D) Tortuous aorta

Ans: C Feedback: While a carotid artery aneurysm is a possibility, it is more likely due to kinking of the carotid artery in this patient with HTN. A tortuous aorta will sometimes cause elevation of the left jugular vein by impairing drainage within the thorax.

A 28-year-old musician comes to your clinic, complaining of a "spot" on his penis. He states his partner noticed it 2 days ago and it hasn't gone away. He says it doesn't hurt. He has had no burning with urination and no pain during intercourse. He has had several partners in the last year and uses condoms occasionally. His past medical history consists of nongonococcal urethritis from Chlamydia and prostatitis. He denies any surgeries. He smokes two packs of cigarettes a day, drinks a case of beer a week, and smokes marijuana and occasionally crack. He has injected IV drugs before but not in the last few years. He is single and currently unemployed. His mother has rheumatoid arthritis and he doesn't know anything about his father. On examination you see a young man appearing deconditioned but pleasant. His vital signs are unremarkable. On visualization of his penis there is a 6-mm red, oval ulcer with an indurated base just proximal to the corona. There is no prepuce because of neonatal circumcision. On palpation the ulcer is nontender. In the inguinal region there is nontender lymphadenopathy. What disorder of the penis is most likely the diagnosis? A) Condylomata acuminata B) Genital herpes C) Syphilitic chancre D) Penile carcinoma

Ans: C Feedback: Primary syphilis causes a larger ulcer that is firm and painless. Syphilis is fairly uncommon but does occur in the highly promiscuous population, especially when coupled with illegal drug use. You should consider further questions and workup regarding HIV status.

A tender, painful swelling of the scrotum should suggest which of the following? A) Acute epididymitis B) Strangulated inguinal hernia C) Torsion of the spermatic cord D) All of the above

Ans: D Feedback: A tender, painful swelling of the scrotum can be a medical emergency. All of these conditions should be considered, as well as acute orchitis.

Which is true of prostate cancer? A) It is commonly lethal. B) It is one of the less common forms of cancer. C) Family history does not appear to be a risk factor. D) Ethnicity is a risk factor.

Ans: D Feedback: Although prostate cancer is the most commonly diagnosed cancer in men, biologic risk and mortality are only 3%. Lung and colon cancers are more common causes of mortality. Genetics appear to account for 42% of cases in one study. The rate of prostate cancer is almost double in African-American men, which is one of the reasons to begin screening at 40 years of age rather than the standard recommendation of 50.

A 26-year-old woman comes to your clinic, complaining of leakage of stool despite generally normal, pain-free bowel movements. She denies any blood in her stool or on the toilet paper. She has had no recent episodes of diarrhea. Her past medical history includes a spontaneous vaginal delivery 3 months ago. She had a fourth-degree tear of the perineal area (from the vagina through the rectum) that was surgically repaired after delivery. A few days later the patient developed an abscess in the anal area that had to be incised and drained. She denies using any tobacco, alcohol, or illegal drugs. Her mother and father are both in good health. She denies any weight gain, weight loss, fever, or night sweats. She is still breastfeeding without any problems. On examination, you visualize a small opening anterior to the anus with some surrounding erythema. There is not a mass or other inflammation on inspection. Digital rectal examination reveals smooth rectal walls with no blood. She has no pain during the rectal examination. Bimanual vaginal examination is also normal. What anal or rectal disorder is the most likely cause of her symptom? A) Anal fissure B) External hemorrhoids C) Internal hemorrhoids D) Anorectal fistula

Ans: D Feedback: Anorectal fistula can commonly cause a leakage of stool, even when the patient is not having a bowel movement. They are common after infections, especially after trauma to the anal musculature (such as in a fourth-degree perineal tear). With more chronic gastrointestinal symptoms, this finding may lead you to suspect Crohn's disease.

A 45-year-old electrical engineer presents to your clinic, complaining of spots on his scrotum. He first noticed the spots several months ago, and they have gotten bigger. He denies any pain with urination or with sexual intercourse. He has had no fever, night sweats, weight gain, or weight loss. His past medical history consists of a vasectomy 10 years ago and mild obesity. He is on medication for hyperlipidemia. He denies any tobacco or illegal drug use and drinks alcohol socially. His mother has Alzheimer's disease and his father died of leukemia. On examination he appears relaxed and has unremarkable vital signs. On visualization of his penis, he is circumcised and has no lesions on his penis. Visualization of his scrotum shows three yellow nodules 2-3 millimeters in diameter. During palpation they are firm and nontender. What abnormality of the male genitalia is this most likely to be? A) Condylomata acuminata B) Syphilitic chancre C) Peyronie's disease D) Epidermoid cysts

Ans: D Feedback: Epidermoid cysts are firm, yellowish, painless cysts on the scrotal skin. They are very common and are benign.

Which of the following is commonly seen in aging men? A) Erectile dysfunction in 20% of all men B) Testicles ride higher within the scrotum C) Strong response to visual erotic cues D) Persistent sexual interest

Ans: D Feedback: Erectile dysfunction affects about half of elderly men but sexual interest generally remains intact. A decrease in sexual interest may indicate other problems such as depression. Visual cues become less important and tactile stimulation more important. The testicles are positioned lower in the scrotum.

Francis is a middle-aged man who noted right-sided lower abdominal pain after straining with yard work. Which of the following would make a hernia more likely? A) Absence of pain with straining B) Absence of bowel sounds in the scrotum C) Absence of a varicocele D) Absence of symmetry of the inguinal areas with straining

Ans: D Feedback: Even in the presence of a hernia, absolute symmetry to inspection may be preserved. The action of straining and increasing intra-abdominal pressure causes the hernia to protrude. Hernias will not necessarily be present on CT scans either unless this maneuver is undertaken. Pain with straining and bowel sounds heard in the scrotum further support the diagnosis of indirect hernia

Claire's daughter brings her in today after Claire fell at her home. Which assessments are indicated at this time? A) Orthostatic vital signs B) Review of her medications C) Assessment of gait and balance D) All of the above

Ans: D Feedback: Falls are common in the elderly and can often result in serious injuries. When assessing the cause of falls, gait and balance should be checked first. Medication, particularly use of more than three, is associated with falls. Vision problems, lower-limb joint problems, and cardiovascular problems such as arrhythmias may be reasonable to search for. Orthostatic vital sign changes should be sought.

Important techniques in performing the rectal examination include which of the following? A) Lubrication B) Waiting for the sphincter to relax C) Explaining what the patient should expect with each step before it occurs D) All of the above

Ans: D Feedback: Lubricating the entire finger (yet removing excess lubricant), being patient while the anal sphincter relaxes, and preparing the patient for each step are key parts of a good rectal examination. The examination itself, while it may be awkward for a patient, should never cause pain in a normal person.

Which of the following accompanies decreased ovarian function? A) Increased sleep B) Diminution of sexual interest C) Enlargement of the clitoris D) Decrease in vaginal secretions

Ans: D Feedback: Menopause, or the cessation of menses for 1 year, commonly occurs in the late 40s to early 50s. Many experience hot flashes, sweating, chills, anxiety, decreased sleep, and urge incontinence. Dyspareunia is common secondary to decreased vaginal secretions. Sexual interest does not normally decrease. The clitoris and length of the vaginal vault decrease in response to decreased estrogen.

Which of the following represents metrorrhagia? A) Fewer than 21 days between menses B) Excessive flow C) Infrequent bleeding D) Bleeding between periods

Ans: D Feedback: Metrorrhagia is bleeding between periods. Menorrhagia is excessive bleeding with menses, while oligomenorrhea is infrequent menses. Polymenorrhea is menstruation with fewer than 21 days between periods.

A 50-year-old truck driver comes to your clinic for a work physical. He has had no upper respiratory, cardiac, pulmonary, gastrointestinal, urinary, or musculoskeletal system complaints. His past medical history is significant for mild arthritis and prior knee surgery in college. He is married and just changed jobs, working for a different trucking company. He smokes one pack of cigarettes a day, drinks less than six beers a week, and denies using any illegal drugs. His mother has high blood pressure and arthritis and his father died of lung cancer in his 60s. On examination, his blood pressure is 130/80 and his pulse is 80. His cardiac, lung and abdominal examinations are normal. He has no inguinal hernia, but on his digital rectal examination, you palpate a soft, smooth, non-tender pedunculated mass on the posterior wall of the rectum. What anal, rectal, or prostate disorder best fits his presentation? A) Internal hemorrhoid B) Prostate cancer C) Anorectal cancer D) Rectal polyp

Ans: D Feedback: Polyps are generally symptom-free and can be found on routine rectal examinations. Proctoscopy with biopsy is necessary to see if the polyp has any potential to become cancerous. Anyone with a rectal polyp needs a colonoscopy.

Which of the following is true of the HPV vaccine? A) Ideally it should be administered within 3 years of first intercourse. B) It covers against almost every HPV type. C) It can be used as adjuvant therapy in cervical cancer. D) It can protect against anogenital lesions.

Ans: D Feedback: The HPV vaccine confers the greatest protection if given before exposure to the HPV. Currently, HPV types 6, 11, 16, and 18 are targeted because these are among the most common types causing cervical cancer. There are many other types, some of which are associated with cervical cancer. It is not recommended for treatment of preexisting cervical cancer but is protective against anogenital lesions.

Which of the following lymph node groups is most commonly involved in breast cancer? A) Lateral B) Subscapular C) Pectoral D) Central

Ans: D Feedback: The central nodes at the apex of the axilla are most commonly involved in breast cancer. The axilla can be viewed roughly as a four-sided pyramid. An examination covering all sides and the apex is unlikely to miss a significant node.

An 89-year-old retired school principal comes for an annual check-up. She would like to know whether or not she should undergo a screening colonoscopy. She has never done this before. Which of the following factors should not be considered when discussing whether she should go for this screening test? A) Life expectancy B) Time interval until benefit from screening accrues C) Patient preference D) Current age of patient

Ans: D Feedback: The current age of the patient is not as important as her actual life expectancy and current health status.

Which of the following changes are expected in vision as part of the normal aging process? A) Cataracts B) Glaucoma C) Macular degeneration D) Blurring of near vision

Ans: D Feedback: The lens loses its elasticity over time as part of the normal aging process, and the eye is less able to accommodate and focus on near objects; therefore, the patient will be expected to have blurring of near vision.

A 70-year-old retired auto mechanic comes to your office because his neighbor is concerned about his memory. The patient himself admits to misplacing his keys more often and forgets what he is supposed to buy from the grocery store and where he has parked the car. He denies getting lost in familiar places. Upon further questioning, he states that his wife of 40 years died 8 months ago; his three children live in three different states; and he has limited his activities because the people he interacted with were "his wife's friends, not his." He drinks a six-pack of beer daily; he does not smoke or use illicit drugs. You perform a mini-mental state examination and obtain a total score of 24 out of 28. Based on this information, what is your most likely diagnosis? A) Benign forgetfulness B) Dementia C) Meningitis D) Depression

Ans: D Feedback: The patient has symptoms of depression: His wife died, he has no real social support system, and he has isolated himself from his usual activities. He also drinks a considerable amount of alcohol on a daily basis, which can further depress his mood. Depression can masquerade as dementia in the elderly and must be considered in a patient with memory loss.

On routine screening you notice that the cup-to-disc ratio of the patient's right eye is 1:2. What ocular condition should you suspect? A) Macular degeneration B) Diabetic retinopathy C) Hypertensive retinopathy D) Glaucoma

Ans: D Feedback: This cup-to-disc ratio means that the cup takes up 50% of the disc, which is abnormally large. This is usually an indication of glaucoma, which is a common cause of visual loss in the elderly. The cup-to-disc changes are not seen in diabetes, hypertension, or macular degeneration. Many elderly do not have regular eye examinations and are not screened for glaucoma.

An 85-year-old retired teacher comes to your office for evaluation of weakness. You obtain a complete history, perform a thorough physical examination, and order laboratory tests. You diagnose her with hyperthyroidism. Based on her age, which of the atypical symptoms of hyperthyroidism is more likely to be seen? A) Fatigue B) Weight loss C) Tachycardia D) Anorexia

Ans: D Feedback: This is an atypical symptom of hyperthyroidism that is more likely to be seen in the older patient.

You are examining a newborn and note that the right testicle is not in the scrotum. What should you do next? A) Refer to urology B) Recheck in 6 months C) Tell the parent the testicle is absent but that this should not affect fertility D) Attempt to bring down the testis from the inguinal canal

Ans: D Feedback: This is not an uncommon finding, and the testis must often be "milked" into the scrotum from the inguinal canal. Six months is too long to wait, but urology referral is unnecessary unless the testicle cannot be brought into the scrotum. An intra-abdominal testis is at much higher risk for testicular cancer.

A 75-year-old retired construction worker comes to your clinic, complaining of bright red blood in the toilet for the last several months. He has no pain with defecation but has occasional constipation. He states he eats a healthy diet with fruits and vegetables and walks 2 miles a day. He has had a 10-pound weight loss over the last 3 months. He denies fever or night sweats. His medical history includes high blood pressure, coronary artery disease, and arthritis. He has also had an appendectomy. He smoked for 40 years, two packs a day, but quit 15 years ago. He used to drink alcohol but doesn't now. His father died in his 60s of a heart attack and his mother had breast cancer in her 70s. On examination he appears his stated age and sits comfortably on the examining table. His blood pressure is 150/85 and his pulse is 88. He is afebrile. His cardiac, lung, and abdominal examinations are normal. Visualization of the anus shows no erythema, masses, or inflammation. Digital rectal examination elicits an irregular, firm mass on the posterior side of the rectum. After you remove your finger you notice frank blood on your glove. What anal or rectal disorder is this patient most likely to have? A) Anal fissure B) Internal hemorrhoid C) Prostate cancer D) Anorectal cancer

Ans: D Feedback: This patient has the common symptom of bright red blood in the toilet over time. He also has had weight loss and has an irregular hard mass in the rectum. It is not uncommon for these masses to be friable (bleed easily), even with gentle manipulation.

Abby is a newly married woman who is unable to have intercourse because of vaginismus. Which of the following is true? A) This is most likely due to lack of lubrication. B) This is most likely due to atrophic vaginitis. C) This is most likely due to pressure on an ovary. D) Psychosocial reasons may cause this condition.

Ans: D Feedback: Vaginismus is an involuntary contraction of the muscles around the vaginal opening. While all of the above may contribute to vaginismus, the psychosocial history must be obtained and frequently is helpful in finding the underlying cause.

A 32-year-old white male comes to your clinic, complaining of aching on the right side of his testicle. He has felt this aching for several months. He states that as the day progresses the aching increases, but when he wakes up in the morning he is pain-free. He denies any pain with urination and states that the pain doesn't change with sexual activity. He denies any fatigue, weight gain, weight loss, fever, or night sweats. His past medical history is unremarkable. He is a married hospital administrator with two children. He notes that he and his wife have been trying to have another baby this year but have so far been unsuccessful despite frequent intercourse. He denies using tobacco, alcohol, or illegal drugs. His father has high blood pressure but his mother is healthy. On examination you see a young man appearing his stated age with unremarkable vital signs. On visualization of his penis, he is circumcised with no lesions. He has no scars along his inguinal area, and palpation of the area shows no lymphadenopathy. On palpation of his scrotum you feel testes with no discrete masses. Upon placing your finger through the right inguinal ring you feel what seems like a bunch of spaghetti. Asking him to bear down, you feel no bulges. The left inguinal ring is unremarkable, with no bulges on bearing down. His prostate examination is unremarkable. What abnormality of the scrotum does he most likely have? A) Hydrocele B) Scrotal hernia C) Scrotal edema D) Varicocele

Ans: D Feedback: Varicoceles are varicose veins surrounding the spermatic cord, coming through the inguinal ring. These veins feel like spaghetti and are often referred to as a "bag of worms." The increased number of veins affects the temperature of the testes, often causing infertility problems. Like most varicose veins in any area, varicoceles can cause a nonspecific aching. Although usually benign, a unilateral varicocele on the right or a varicocele which does not resolve in the supine position deserves further workup.

A 55-year-old married homemaker comes to your clinic, complaining of 6 months of vaginal itching and discomfort with intercourse. She has not had a discharge and has had no pain with urination. She has not had a period in over 2 years. She has no other symptoms. Her past medical history consists of the removal of her gallbladder. She denies the use of tobacco, alcohol, and illegal drugs. Her mother has breast cancer and her father has coronary artery disease, high blood pressure, and Alzheimer's disease. On examination, she appears healthy and has unremarkable vital signs. There is no lymphadenopathy with palpation of the inguinal nodes. Visualization of the vulva shows dry skin but no lesions or masses. The labia are somewhat smaller than usual. Speculum examination reveals scant discharge and the vaginal walls are red, dry, and bleed easily. Bimanual examination is unremarkable. The KOH whiff test produces no unusual odor and there are no clue cells on the wet prep. What form of vaginitis is this patient most likely to have? A) Trichomonas vaginitis B) Candida vaginitis C) Bacterial vaginosis D) Atrophic vaginitis

Ans: D Feedback: The itching and pain with intercourse in atrophic vaginitis are due to the decreased amount of estrogen after menopause. There is generally scant discharge and the wet prep and KOH whiff test are unremarkable. Use of vaginal lubricants or hormonal replacement in selected patients often corrects the problem.

A 42-year-old realtor comes to your clinic, complaining of "growths" in her vulvar area. She is currently undergoing a divorce and is convinced she has a sexually transmitted disease. She denies any vaginal discharge or pain with urination. She has had no fever, malaise, or night sweats. Her past medical history consists of depression and hypothyroidism. She has had two spontaneous vaginal deliveries and one cesarean section. She has had no other surgeries. She denies smoking or drug use. She has two to three drinks weekly. Her mother also has hypothyroidism and her father has high blood pressure and hypercholesterolemia. On examination, you see a woman who is anxious but appears otherwise healthy. Her blood pressure, pulse, and temperature are unremarkable. On the visualization of the perineum, you see two 2- to 3-mm, round, yellow nodules on the left labia. On palpation, they are nontender and quite firm. What diagnosis best fits this description of her examination? A) Genital herpes B) Condylomata acuminata C) Syphilitic chancre D) Epidermoid cyst

Ans: D Feedback: These cysts are small, firm, round cystic nodules in the labia that are nonpainful. These do not represent a sexually transmitted infection, but rather a blocked sebaceous gland.


Kaugnay na mga set ng pag-aaral

Adult Development Module 1 Chapter 1-2

View Set

insurance life simulated test questions

View Set

Ophthalmic Prisms and Prismatic Effects

View Set

The Enlightenment Assignment and Quiz

View Set

Concept 2 - Recognizing Literary Terms in Examples

View Set

Chapter 21 - Conflict, Workplace Violence, and Negotiation LOOK AT HIGHLIGHTS IN BOOK

View Set

Chapter 34- Caring for the Special Needs Child

View Set

Entrepreneurial Leadership Exam-3 Cole

View Set